Prove that $frac6(a^2 + b^2 + c^2)a + b + c geq frac(a + b)^2b + c + frac(b + c)^2c + a + frac(a + c)^2a + b$

The name of the pictureThe name of the pictureThe name of the pictureClash Royale CLAN TAG#URR8PPP











up vote
1
down vote

favorite
1












Prove that if $a,b,c$ are the lengths of the edges of a given triangle, then the following inequality holds:



$frac6(a^2 + b^2 + c^2)a + b + c geq frac(a + b)^2b + c + frac(b + c)^2c + a + frac(a + c)^2a + b$.



I have tried so far using the Ravi substitution, but the expression got even uglier. I have also tried making the substitutions: $x = a + b$, $y = b + c$, $z = a + c$, and after using $3(a^2 + b^2 + c^2) geq (a + b + c)^2$, this resulted in showing that: $x + y + z geq fracx^2y + fracy^2z + fracz^2x$, where we also know (from triangle inequality) that: $z geq fracx+y3$ and the other 2 analogues, but at this point I got stuck.







share|cite|improve this question



















  • This question is missing important information: the source and motivation of the problem. You can edit the post to add additional information. For advice on writing a good question, see math.meta.stackexchange.com/questions/9959/…
    – Carl Mummert
    Aug 7 at 17:35














up vote
1
down vote

favorite
1












Prove that if $a,b,c$ are the lengths of the edges of a given triangle, then the following inequality holds:



$frac6(a^2 + b^2 + c^2)a + b + c geq frac(a + b)^2b + c + frac(b + c)^2c + a + frac(a + c)^2a + b$.



I have tried so far using the Ravi substitution, but the expression got even uglier. I have also tried making the substitutions: $x = a + b$, $y = b + c$, $z = a + c$, and after using $3(a^2 + b^2 + c^2) geq (a + b + c)^2$, this resulted in showing that: $x + y + z geq fracx^2y + fracy^2z + fracz^2x$, where we also know (from triangle inequality) that: $z geq fracx+y3$ and the other 2 analogues, but at this point I got stuck.







share|cite|improve this question



















  • This question is missing important information: the source and motivation of the problem. You can edit the post to add additional information. For advice on writing a good question, see math.meta.stackexchange.com/questions/9959/…
    – Carl Mummert
    Aug 7 at 17:35












up vote
1
down vote

favorite
1









up vote
1
down vote

favorite
1






1





Prove that if $a,b,c$ are the lengths of the edges of a given triangle, then the following inequality holds:



$frac6(a^2 + b^2 + c^2)a + b + c geq frac(a + b)^2b + c + frac(b + c)^2c + a + frac(a + c)^2a + b$.



I have tried so far using the Ravi substitution, but the expression got even uglier. I have also tried making the substitutions: $x = a + b$, $y = b + c$, $z = a + c$, and after using $3(a^2 + b^2 + c^2) geq (a + b + c)^2$, this resulted in showing that: $x + y + z geq fracx^2y + fracy^2z + fracz^2x$, where we also know (from triangle inequality) that: $z geq fracx+y3$ and the other 2 analogues, but at this point I got stuck.







share|cite|improve this question











Prove that if $a,b,c$ are the lengths of the edges of a given triangle, then the following inequality holds:



$frac6(a^2 + b^2 + c^2)a + b + c geq frac(a + b)^2b + c + frac(b + c)^2c + a + frac(a + c)^2a + b$.



I have tried so far using the Ravi substitution, but the expression got even uglier. I have also tried making the substitutions: $x = a + b$, $y = b + c$, $z = a + c$, and after using $3(a^2 + b^2 + c^2) geq (a + b + c)^2$, this resulted in showing that: $x + y + z geq fracx^2y + fracy^2z + fracz^2x$, where we also know (from triangle inequality) that: $z geq fracx+y3$ and the other 2 analogues, but at this point I got stuck.









share|cite|improve this question










share|cite|improve this question




share|cite|improve this question









asked Aug 6 at 19:21









Sandel

263




263











  • This question is missing important information: the source and motivation of the problem. You can edit the post to add additional information. For advice on writing a good question, see math.meta.stackexchange.com/questions/9959/…
    – Carl Mummert
    Aug 7 at 17:35
















  • This question is missing important information: the source and motivation of the problem. You can edit the post to add additional information. For advice on writing a good question, see math.meta.stackexchange.com/questions/9959/…
    – Carl Mummert
    Aug 7 at 17:35















This question is missing important information: the source and motivation of the problem. You can edit the post to add additional information. For advice on writing a good question, see math.meta.stackexchange.com/questions/9959/…
– Carl Mummert
Aug 7 at 17:35




This question is missing important information: the source and motivation of the problem. You can edit the post to add additional information. For advice on writing a good question, see math.meta.stackexchange.com/questions/9959/…
– Carl Mummert
Aug 7 at 17:35










2 Answers
2






active

oldest

votes

















up vote
0
down vote













Computer assisted proof.



The sides of the triangle can be cyclically rearranged, so that $a$ is the smallest side. Then the other two sides are of the shape $a+s$, $a+s+t$ with $0le s$ and $0le t<a$. (The last inequality, $t< a$, insures that the biggest side, $a+s+t$, is $< a+(a+s)$.



We have then to consider the two cases, where $a+s$, and $a+s+t$ are either $b,c$, or $c,b$.



We compute the difference of the quantities in the OP:



sage: var('a,b,c,s,t');
sage: f(a,b,c) = (a+b)^2 / (b+c)
sage: E(a,b,c) = 6*(a^2+b^2+c^2)/(a+b+c) - f(a,b,c) - f(b,c,a) - f(c,a,b)
sage: G(a,b,c) = E(a,b,c).numerator()

sage: G(a,a+s,a+s+t).expand()
8*a^3*s^2 + 18*a^2*s^3 + 11*a*s^4 + 2*s^5 + 8*a^3*s*t
+ 36*a^2*s^2*t + 34*a*s^3*t + 9*s^4*t + 8*a^3*t^2 + 30*a^2*s*t^2
+ 33*a*s^2*t^2 + 10*s^3*t^2 + 6*a^2*t^3 + 10*a*s*t^3 + 3*s^2*t^3
- a*t^4 - 2*s*t^4 - t^5

sage: G(a,a+s+t,a+s).expand()
8*a^3*s^2 + 18*a^2*s^3 + 11*a*s^4 + 2*s^5 + 8*a^3*s*t
+ 18*a^2*s^2*t + 10*a*s^3*t + s^4*t + 8*a^3*t^2 + 12*a^2*s*t^2
- 3*a*s^2*t^2 - 6*s^3*t^2
+ 6*a^2*t^3
- 2*a*s*t^3 - 7*s^2*t^3 - a*t^4 - 4*s*t^4 - t^5


(Sage code was inserted. Slightly rearranged to fit in the window.)



So we have to dominate the minus terms by using the plus terms.



In the first case things are simple, we have for instance $8a^3t^2 -at^4-t^5>0$, and $8a^3st-2st^4>0$.



In the second case we have more to type, there is no intelligence involved more than matching the degrees of $s$ while covering the minus terms with the plus terms:
$$
beginaligned
8 , a^3 s^2 + 18 , a^2 s^2 t - 3 , a s^2 t^2 - 7 , s^2 t^3 &ge 0 ,\
18 , a^2 s^3 - 6 , s^3 t^2 &ge 0 ,\
8 , a^3 s t - 2 , a s t^3 - 4 , s t^4 &ge 0 ,\
8 , a^3 t^2 + 6 , a^2 t^3 - a t^4 - t^5 &>0 .
endaligned
$$
$square$




Personal note: The given proof misses by intention any beauty, since the author of the problem also tried desperately to offer an inequality without symmetry, without a visual possibility of a Chebyshev rearrangement, and with the $ge$ sign on the side that does not allow an application of a Cauchy inequality, so that things may become symmetric after some steps. I hope (s)he may enjoy this proof, at least as i enjoyed typing it.






share|cite|improve this answer




























    up vote
    0
    down vote













    Let $c=maxa,b,c$, $a=x+u$, $b=x+v$ and $c=x+u+v$, where $x>0$, $ugeq0$ and $vgeq0.$



    Thus, we need to prove that
    $$8(u^2-uv+v^2)x^3+6(3u^3-2u^2v+uv^2+3v^3)x^2+$$
    $$+(11u^4+2u^3v-15u^2v^2+26uv^3+11v^4)x+2(u^5+u^4v-3u^3v^2+u^2v^2+5uv^4+v^5)geq0,$$
    which is obvious.



    Also, the $uvw$'s technique helps, but it's very complicated here.






    share|cite|improve this answer























      Your Answer




      StackExchange.ifUsing("editor", function ()
      return StackExchange.using("mathjaxEditing", function ()
      StackExchange.MarkdownEditor.creationCallbacks.add(function (editor, postfix)
      StackExchange.mathjaxEditing.prepareWmdForMathJax(editor, postfix, [["$", "$"], ["\\(","\\)"]]);
      );
      );
      , "mathjax-editing");

      StackExchange.ready(function()
      var channelOptions =
      tags: "".split(" "),
      id: "69"
      ;
      initTagRenderer("".split(" "), "".split(" "), channelOptions);

      StackExchange.using("externalEditor", function()
      // Have to fire editor after snippets, if snippets enabled
      if (StackExchange.settings.snippets.snippetsEnabled)
      StackExchange.using("snippets", function()
      createEditor();
      );

      else
      createEditor();

      );

      function createEditor()
      StackExchange.prepareEditor(
      heartbeatType: 'answer',
      convertImagesToLinks: true,
      noModals: false,
      showLowRepImageUploadWarning: true,
      reputationToPostImages: 10,
      bindNavPrevention: true,
      postfix: "",
      noCode: true, onDemand: true,
      discardSelector: ".discard-answer"
      ,immediatelyShowMarkdownHelp:true
      );



      );








       

      draft saved


      draft discarded


















      StackExchange.ready(
      function ()
      StackExchange.openid.initPostLogin('.new-post-login', 'https%3a%2f%2fmath.stackexchange.com%2fquestions%2f2874232%2fprove-that-frac6a2-b2-c2a-b-c-geq-fraca-b2b-c%23new-answer', 'question_page');

      );

      Post as a guest






























      2 Answers
      2






      active

      oldest

      votes








      2 Answers
      2






      active

      oldest

      votes









      active

      oldest

      votes






      active

      oldest

      votes








      up vote
      0
      down vote













      Computer assisted proof.



      The sides of the triangle can be cyclically rearranged, so that $a$ is the smallest side. Then the other two sides are of the shape $a+s$, $a+s+t$ with $0le s$ and $0le t<a$. (The last inequality, $t< a$, insures that the biggest side, $a+s+t$, is $< a+(a+s)$.



      We have then to consider the two cases, where $a+s$, and $a+s+t$ are either $b,c$, or $c,b$.



      We compute the difference of the quantities in the OP:



      sage: var('a,b,c,s,t');
      sage: f(a,b,c) = (a+b)^2 / (b+c)
      sage: E(a,b,c) = 6*(a^2+b^2+c^2)/(a+b+c) - f(a,b,c) - f(b,c,a) - f(c,a,b)
      sage: G(a,b,c) = E(a,b,c).numerator()

      sage: G(a,a+s,a+s+t).expand()
      8*a^3*s^2 + 18*a^2*s^3 + 11*a*s^4 + 2*s^5 + 8*a^3*s*t
      + 36*a^2*s^2*t + 34*a*s^3*t + 9*s^4*t + 8*a^3*t^2 + 30*a^2*s*t^2
      + 33*a*s^2*t^2 + 10*s^3*t^2 + 6*a^2*t^3 + 10*a*s*t^3 + 3*s^2*t^3
      - a*t^4 - 2*s*t^4 - t^5

      sage: G(a,a+s+t,a+s).expand()
      8*a^3*s^2 + 18*a^2*s^3 + 11*a*s^4 + 2*s^5 + 8*a^3*s*t
      + 18*a^2*s^2*t + 10*a*s^3*t + s^4*t + 8*a^3*t^2 + 12*a^2*s*t^2
      - 3*a*s^2*t^2 - 6*s^3*t^2
      + 6*a^2*t^3
      - 2*a*s*t^3 - 7*s^2*t^3 - a*t^4 - 4*s*t^4 - t^5


      (Sage code was inserted. Slightly rearranged to fit in the window.)



      So we have to dominate the minus terms by using the plus terms.



      In the first case things are simple, we have for instance $8a^3t^2 -at^4-t^5>0$, and $8a^3st-2st^4>0$.



      In the second case we have more to type, there is no intelligence involved more than matching the degrees of $s$ while covering the minus terms with the plus terms:
      $$
      beginaligned
      8 , a^3 s^2 + 18 , a^2 s^2 t - 3 , a s^2 t^2 - 7 , s^2 t^3 &ge 0 ,\
      18 , a^2 s^3 - 6 , s^3 t^2 &ge 0 ,\
      8 , a^3 s t - 2 , a s t^3 - 4 , s t^4 &ge 0 ,\
      8 , a^3 t^2 + 6 , a^2 t^3 - a t^4 - t^5 &>0 .
      endaligned
      $$
      $square$




      Personal note: The given proof misses by intention any beauty, since the author of the problem also tried desperately to offer an inequality without symmetry, without a visual possibility of a Chebyshev rearrangement, and with the $ge$ sign on the side that does not allow an application of a Cauchy inequality, so that things may become symmetric after some steps. I hope (s)he may enjoy this proof, at least as i enjoyed typing it.






      share|cite|improve this answer

























        up vote
        0
        down vote













        Computer assisted proof.



        The sides of the triangle can be cyclically rearranged, so that $a$ is the smallest side. Then the other two sides are of the shape $a+s$, $a+s+t$ with $0le s$ and $0le t<a$. (The last inequality, $t< a$, insures that the biggest side, $a+s+t$, is $< a+(a+s)$.



        We have then to consider the two cases, where $a+s$, and $a+s+t$ are either $b,c$, or $c,b$.



        We compute the difference of the quantities in the OP:



        sage: var('a,b,c,s,t');
        sage: f(a,b,c) = (a+b)^2 / (b+c)
        sage: E(a,b,c) = 6*(a^2+b^2+c^2)/(a+b+c) - f(a,b,c) - f(b,c,a) - f(c,a,b)
        sage: G(a,b,c) = E(a,b,c).numerator()

        sage: G(a,a+s,a+s+t).expand()
        8*a^3*s^2 + 18*a^2*s^3 + 11*a*s^4 + 2*s^5 + 8*a^3*s*t
        + 36*a^2*s^2*t + 34*a*s^3*t + 9*s^4*t + 8*a^3*t^2 + 30*a^2*s*t^2
        + 33*a*s^2*t^2 + 10*s^3*t^2 + 6*a^2*t^3 + 10*a*s*t^3 + 3*s^2*t^3
        - a*t^4 - 2*s*t^4 - t^5

        sage: G(a,a+s+t,a+s).expand()
        8*a^3*s^2 + 18*a^2*s^3 + 11*a*s^4 + 2*s^5 + 8*a^3*s*t
        + 18*a^2*s^2*t + 10*a*s^3*t + s^4*t + 8*a^3*t^2 + 12*a^2*s*t^2
        - 3*a*s^2*t^2 - 6*s^3*t^2
        + 6*a^2*t^3
        - 2*a*s*t^3 - 7*s^2*t^3 - a*t^4 - 4*s*t^4 - t^5


        (Sage code was inserted. Slightly rearranged to fit in the window.)



        So we have to dominate the minus terms by using the plus terms.



        In the first case things are simple, we have for instance $8a^3t^2 -at^4-t^5>0$, and $8a^3st-2st^4>0$.



        In the second case we have more to type, there is no intelligence involved more than matching the degrees of $s$ while covering the minus terms with the plus terms:
        $$
        beginaligned
        8 , a^3 s^2 + 18 , a^2 s^2 t - 3 , a s^2 t^2 - 7 , s^2 t^3 &ge 0 ,\
        18 , a^2 s^3 - 6 , s^3 t^2 &ge 0 ,\
        8 , a^3 s t - 2 , a s t^3 - 4 , s t^4 &ge 0 ,\
        8 , a^3 t^2 + 6 , a^2 t^3 - a t^4 - t^5 &>0 .
        endaligned
        $$
        $square$




        Personal note: The given proof misses by intention any beauty, since the author of the problem also tried desperately to offer an inequality without symmetry, without a visual possibility of a Chebyshev rearrangement, and with the $ge$ sign on the side that does not allow an application of a Cauchy inequality, so that things may become symmetric after some steps. I hope (s)he may enjoy this proof, at least as i enjoyed typing it.






        share|cite|improve this answer























          up vote
          0
          down vote










          up vote
          0
          down vote









          Computer assisted proof.



          The sides of the triangle can be cyclically rearranged, so that $a$ is the smallest side. Then the other two sides are of the shape $a+s$, $a+s+t$ with $0le s$ and $0le t<a$. (The last inequality, $t< a$, insures that the biggest side, $a+s+t$, is $< a+(a+s)$.



          We have then to consider the two cases, where $a+s$, and $a+s+t$ are either $b,c$, or $c,b$.



          We compute the difference of the quantities in the OP:



          sage: var('a,b,c,s,t');
          sage: f(a,b,c) = (a+b)^2 / (b+c)
          sage: E(a,b,c) = 6*(a^2+b^2+c^2)/(a+b+c) - f(a,b,c) - f(b,c,a) - f(c,a,b)
          sage: G(a,b,c) = E(a,b,c).numerator()

          sage: G(a,a+s,a+s+t).expand()
          8*a^3*s^2 + 18*a^2*s^3 + 11*a*s^4 + 2*s^5 + 8*a^3*s*t
          + 36*a^2*s^2*t + 34*a*s^3*t + 9*s^4*t + 8*a^3*t^2 + 30*a^2*s*t^2
          + 33*a*s^2*t^2 + 10*s^3*t^2 + 6*a^2*t^3 + 10*a*s*t^3 + 3*s^2*t^3
          - a*t^4 - 2*s*t^4 - t^5

          sage: G(a,a+s+t,a+s).expand()
          8*a^3*s^2 + 18*a^2*s^3 + 11*a*s^4 + 2*s^5 + 8*a^3*s*t
          + 18*a^2*s^2*t + 10*a*s^3*t + s^4*t + 8*a^3*t^2 + 12*a^2*s*t^2
          - 3*a*s^2*t^2 - 6*s^3*t^2
          + 6*a^2*t^3
          - 2*a*s*t^3 - 7*s^2*t^3 - a*t^4 - 4*s*t^4 - t^5


          (Sage code was inserted. Slightly rearranged to fit in the window.)



          So we have to dominate the minus terms by using the plus terms.



          In the first case things are simple, we have for instance $8a^3t^2 -at^4-t^5>0$, and $8a^3st-2st^4>0$.



          In the second case we have more to type, there is no intelligence involved more than matching the degrees of $s$ while covering the minus terms with the plus terms:
          $$
          beginaligned
          8 , a^3 s^2 + 18 , a^2 s^2 t - 3 , a s^2 t^2 - 7 , s^2 t^3 &ge 0 ,\
          18 , a^2 s^3 - 6 , s^3 t^2 &ge 0 ,\
          8 , a^3 s t - 2 , a s t^3 - 4 , s t^4 &ge 0 ,\
          8 , a^3 t^2 + 6 , a^2 t^3 - a t^4 - t^5 &>0 .
          endaligned
          $$
          $square$




          Personal note: The given proof misses by intention any beauty, since the author of the problem also tried desperately to offer an inequality without symmetry, without a visual possibility of a Chebyshev rearrangement, and with the $ge$ sign on the side that does not allow an application of a Cauchy inequality, so that things may become symmetric after some steps. I hope (s)he may enjoy this proof, at least as i enjoyed typing it.






          share|cite|improve this answer













          Computer assisted proof.



          The sides of the triangle can be cyclically rearranged, so that $a$ is the smallest side. Then the other two sides are of the shape $a+s$, $a+s+t$ with $0le s$ and $0le t<a$. (The last inequality, $t< a$, insures that the biggest side, $a+s+t$, is $< a+(a+s)$.



          We have then to consider the two cases, where $a+s$, and $a+s+t$ are either $b,c$, or $c,b$.



          We compute the difference of the quantities in the OP:



          sage: var('a,b,c,s,t');
          sage: f(a,b,c) = (a+b)^2 / (b+c)
          sage: E(a,b,c) = 6*(a^2+b^2+c^2)/(a+b+c) - f(a,b,c) - f(b,c,a) - f(c,a,b)
          sage: G(a,b,c) = E(a,b,c).numerator()

          sage: G(a,a+s,a+s+t).expand()
          8*a^3*s^2 + 18*a^2*s^3 + 11*a*s^4 + 2*s^5 + 8*a^3*s*t
          + 36*a^2*s^2*t + 34*a*s^3*t + 9*s^4*t + 8*a^3*t^2 + 30*a^2*s*t^2
          + 33*a*s^2*t^2 + 10*s^3*t^2 + 6*a^2*t^3 + 10*a*s*t^3 + 3*s^2*t^3
          - a*t^4 - 2*s*t^4 - t^5

          sage: G(a,a+s+t,a+s).expand()
          8*a^3*s^2 + 18*a^2*s^3 + 11*a*s^4 + 2*s^5 + 8*a^3*s*t
          + 18*a^2*s^2*t + 10*a*s^3*t + s^4*t + 8*a^3*t^2 + 12*a^2*s*t^2
          - 3*a*s^2*t^2 - 6*s^3*t^2
          + 6*a^2*t^3
          - 2*a*s*t^3 - 7*s^2*t^3 - a*t^4 - 4*s*t^4 - t^5


          (Sage code was inserted. Slightly rearranged to fit in the window.)



          So we have to dominate the minus terms by using the plus terms.



          In the first case things are simple, we have for instance $8a^3t^2 -at^4-t^5>0$, and $8a^3st-2st^4>0$.



          In the second case we have more to type, there is no intelligence involved more than matching the degrees of $s$ while covering the minus terms with the plus terms:
          $$
          beginaligned
          8 , a^3 s^2 + 18 , a^2 s^2 t - 3 , a s^2 t^2 - 7 , s^2 t^3 &ge 0 ,\
          18 , a^2 s^3 - 6 , s^3 t^2 &ge 0 ,\
          8 , a^3 s t - 2 , a s t^3 - 4 , s t^4 &ge 0 ,\
          8 , a^3 t^2 + 6 , a^2 t^3 - a t^4 - t^5 &>0 .
          endaligned
          $$
          $square$




          Personal note: The given proof misses by intention any beauty, since the author of the problem also tried desperately to offer an inequality without symmetry, without a visual possibility of a Chebyshev rearrangement, and with the $ge$ sign on the side that does not allow an application of a Cauchy inequality, so that things may become symmetric after some steps. I hope (s)he may enjoy this proof, at least as i enjoyed typing it.







          share|cite|improve this answer













          share|cite|improve this answer



          share|cite|improve this answer











          answered Aug 6 at 20:29









          dan_fulea

          4,2171211




          4,2171211




















              up vote
              0
              down vote













              Let $c=maxa,b,c$, $a=x+u$, $b=x+v$ and $c=x+u+v$, where $x>0$, $ugeq0$ and $vgeq0.$



              Thus, we need to prove that
              $$8(u^2-uv+v^2)x^3+6(3u^3-2u^2v+uv^2+3v^3)x^2+$$
              $$+(11u^4+2u^3v-15u^2v^2+26uv^3+11v^4)x+2(u^5+u^4v-3u^3v^2+u^2v^2+5uv^4+v^5)geq0,$$
              which is obvious.



              Also, the $uvw$'s technique helps, but it's very complicated here.






              share|cite|improve this answer



























                up vote
                0
                down vote













                Let $c=maxa,b,c$, $a=x+u$, $b=x+v$ and $c=x+u+v$, where $x>0$, $ugeq0$ and $vgeq0.$



                Thus, we need to prove that
                $$8(u^2-uv+v^2)x^3+6(3u^3-2u^2v+uv^2+3v^3)x^2+$$
                $$+(11u^4+2u^3v-15u^2v^2+26uv^3+11v^4)x+2(u^5+u^4v-3u^3v^2+u^2v^2+5uv^4+v^5)geq0,$$
                which is obvious.



                Also, the $uvw$'s technique helps, but it's very complicated here.






                share|cite|improve this answer

























                  up vote
                  0
                  down vote










                  up vote
                  0
                  down vote









                  Let $c=maxa,b,c$, $a=x+u$, $b=x+v$ and $c=x+u+v$, where $x>0$, $ugeq0$ and $vgeq0.$



                  Thus, we need to prove that
                  $$8(u^2-uv+v^2)x^3+6(3u^3-2u^2v+uv^2+3v^3)x^2+$$
                  $$+(11u^4+2u^3v-15u^2v^2+26uv^3+11v^4)x+2(u^5+u^4v-3u^3v^2+u^2v^2+5uv^4+v^5)geq0,$$
                  which is obvious.



                  Also, the $uvw$'s technique helps, but it's very complicated here.






                  share|cite|improve this answer















                  Let $c=maxa,b,c$, $a=x+u$, $b=x+v$ and $c=x+u+v$, where $x>0$, $ugeq0$ and $vgeq0.$



                  Thus, we need to prove that
                  $$8(u^2-uv+v^2)x^3+6(3u^3-2u^2v+uv^2+3v^3)x^2+$$
                  $$+(11u^4+2u^3v-15u^2v^2+26uv^3+11v^4)x+2(u^5+u^4v-3u^3v^2+u^2v^2+5uv^4+v^5)geq0,$$
                  which is obvious.



                  Also, the $uvw$'s technique helps, but it's very complicated here.







                  share|cite|improve this answer















                  share|cite|improve this answer



                  share|cite|improve this answer








                  edited Aug 7 at 7:08


























                  answered Aug 7 at 3:09









                  Michael Rozenberg

                  88.2k1579180




                  88.2k1579180






















                       

                      draft saved


                      draft discarded


























                       


                      draft saved


                      draft discarded














                      StackExchange.ready(
                      function ()
                      StackExchange.openid.initPostLogin('.new-post-login', 'https%3a%2f%2fmath.stackexchange.com%2fquestions%2f2874232%2fprove-that-frac6a2-b2-c2a-b-c-geq-fraca-b2b-c%23new-answer', 'question_page');

                      );

                      Post as a guest













































































                      Comments

                      Popular posts from this blog

                      Color the edges and diagonals of a regular polygon

                      Relationship between determinant of matrix and determinant of adjoint?

                      What is the equation of a 3D cone with generalised tilt?